Transformación de Lorentz del estado de vacío

En general, el hamiltoniano H tiene un valor esperado de vacío distinto de cero (VEV):

H | Ω = mi 0 | Ω ,
dónde | Ω es el estado de vacío. La energía de punto cero del oscilador cuántico es un buen ejemplo para tener un VEV que no se desvanece.

El problema que me surge es que, ¿cómo se transforma este estado de vacío bajo la transformación de Lorentz?

Por un lado, podemos pensar que este estado de vacío es la representación trivial del grupo de Poincaré, que es un invariante de Lorentz. (En la clasificación de Wigner, el estado de vacío está en el ( 0 , 0 , 0 , 0 ) clase, si entiendo correctamente.) Así que

tu ( Λ ) | Ω = mi i θ | Ω

Por otro lado, el estado de vacío tiene un impulso 4 distinto de cero: pag = ( mi 0 , 0 ) . Entonces parece que debería transformarse como otros irreps:

tu ( Λ ) | pag , σ = σ C σ σ | Λ pag , σ ,
Ahora, lo importante es, Λ pag podría tener un impulso 3 distinto de cero. Pero el estado de vacío debería tener cero impulso 3. ¡Estallido!

En los libros de texto, un procedimiento común para reconciliar esta contradicción es desplazar el hamiltoniano H H mi 0 ; pero, el álgebra de Poincaré se vería afectada:

[ k i , PAG j ] = d i j H [ k i , PAG j ] = d i j H + d i j mi 0

Gracias.

Son demasiadas preguntas en una. ¿Podría tal vez enfocar la pregunta? También puede hacer algunas de estas como preguntas SE diferentes.
Lo siento, no entiendo. En una QFT relativista, se supone que el estado de vacío es el estado invariante único (por lo tanto, es un estado fundamental) bajo todo el grupo de Poincaré y, en consecuencia, tiene un impulso de cero cuatro.
@V.Moretti: Pero el vacío también es una representación del grupo de Poincaré, que debe transformarse según la regla del grupo de Poincaré. Por lo tanto, si tiene un impulso 4 que no desaparece en un marco, puede adquirir un impulso 3 que no desaparece en otro marco, lo que contradice la invariancia de Lorentz.
El oscilador armónico no es un buen ejemplo: no es relativista y no es invariante de traducción.
@Adán está bien. ¿Qué pasa con la teoría del campo libre, que se considera como la superposición de osciladores cuánticos?
Hay dos nociones de invariancia de Poincaré: (1) en el espacio de Hilbert de la teoría existe una representación unitaria del grupo de Poincaré que transforma covariantemente el álgebra de observables. (2) Además de (1) también existe un estado único que es invariable bajo la repetición 1. En QFT (1) y (2) se suponen ambos.
Un campo cuántico libre no es una superposición de osciladores. Esa imagen es solo heurística, es una representación de Fock de un álgebra de Weyl adecuada.
El vacío en QFT relativista es la representación trivial del grupo de Poincaré, por lo que no tiene cuatro impulsos. En realidad, para probarlo, es suficiente que el estado sea invariante bajo cuatro traslaciones (y se supone que sí).
@V.Moretti: Creo que el objetivo del OP es que si comienza con una teoría libre, puede diagonalizar esta teoría en el espacio de momento, donde solo tiene una colección de osciladores armónicos. Entonces encuentras que el vacío tiene una energía de punto cero. k ω k / 2 . (Esa es la energía del vacío que da lugar al famoso error de orden de magnitud de 120 para la "predicción" de la constante cosmológica).
@V.Moretti: ¿qué pasa con el estado fundamental de una teoría interactiva? ¿Se supone que también es invariante bajo traducción o se puede probar de alguna manera? Consulte mi pregunta: physics.stackexchange.com/questions/99920/…
Se asume también para las teorías que interactúan.

Respuestas (1)

Como se ha mencionado en los comentarios, se supone que el QFT tiene un estado de vacío que es aniquilado por PAG m . Este es realmente un punto muy importante, ya que es una de las diferencias cruciales entre la QFT de espacio plano y la QFT de espacio-tiempo curvo. Esto se explica en el libro QFT in Curved spacetime de Wald. Esencialmente, en las teorías cuánticas, la "estructura cinemática" está fijada por las relaciones de conmutación canónicas entre la posición canónica y los momentos (campos y momentos conjugados en QFT). En QM no relativista hay un número finito de grados de libertad, por lo que el teorema de Stone von-Neumann te dice que hay un espacio de Hilbert único y una elección de operadores en el espacio de Hilbert. El teorema no se cumple para un número infinito de grados de libertad (QFT), y en realidad hay infinitas opciones no equivalentes para el espacio de Hilbert para campos cuánticos. En el espacio-tiempo plano, el requisito de que exista un estado tal que PAG m | 0 = 0 elige un espacio único de Hilbert. En los espacio-tiempos curvos que, en general, no tienen vectores de muerte, no está claro cómo identificar un único espacio de Hilbert, y esta es una gran dificultad en el espacio curvo QFT.

Entonces la respuesta es que se supone PAG m | Ω = 0 y esta es una suposición muy importante.

Aunque el procedimiento de resta que mencionas se usa en muchos libros de texto, no creo que sea realmente la solución correcta. El argumento suele ser que el término de resta no es observable, ya que solo aporta una fase a la matriz s y solo medimos las diferencias de energía, pero esto realmente no se sostiene cuando la gravedad o la supersimetría están en la imagen. De hecho, como usted nota, no está claro que el álgebra de Poincaré se cierre bajo tal resta.

Los operadores en dos teorías con diferentes hamiltonianos no necesitan ser los mismos, solo necesitan cambiar de tal manera que el Álgebra de Poincaré todavía se cierre. Si tuvieras un hamiltoniano original H 0 y lo perturbaste por H 0 H = H 0 + V , luego enviando H H mi 0 básicamente equivale a perturbar el hamiltoniano H 0 por V mi 0 . Para perturbaciones construidas a partir de campos locales de una manera invariante de Lorentz, siempre es posible alterar el álgebra de tal manera que incluso en la teoría de interacción el álgebra se cierra.

De hecho, esta es básicamente la respuesta a la pregunta: ¿Qué tipo de perturbaciones puedo agregar al hamiltoniano que dan como resultado una teoría invariante de Lorentz que obedece al principio de descomposición de conglomerados? Voy a esbozar cómo va el argumento.

En la teoría libre tenemos estados de la forma a pag . . . | 0 . Esta es la definición habitual de nuestro espacio libre de Hilbert. Considerar [ k i 0 , H 0 ] = i PAG i . Queremos que la etiqueta de momento signifique lo mismo después de la perturbación (solo alteramos nuestras energías) pero enviamos H 0 H 0 + V . Así que ingenuamente tomamos un término [ k i 0 , V ] . Requerir que estos dos operadores se trasladen es una restricción demasiado fuerte. Entonces, en lugar de eso, permitimos k i 0 k i 0 + k i V tal que

[ k 0 , V ] + [ k V , H 0 ] + [ k V , V ] = 0
Uso de las relaciones de conmutación [ a , a ] , etc. puede demostrar que si V es de la forma
V = d 3 X V ( ϕ ( X ) )
dónde ϕ ( X ) es un campo local construido a partir de los operadores de creación y aniquilación, luego una solución
k i V = d 3 X X i V ( ϕ )
cierra el álgebra (realmente solo necesita verificar el conmutador que menciona). Esto no quiere decir que los QFT locales sean la única forma de construir sistemas cuánticos invariantes de Lorentz, simplemente dice que los QFT locales son algunas de las soluciones.

El problema es ese mi 0 no está construido a partir de campos locales y ni siquiera se puede escribir de manera local en un QFT de volumen infinito. Querrías algo de la forma V = d 3 X mi 0 V . Para tal perturbación k V = 0 y no podemos satisfacer

[ k i V , PAG j ] = d i j mi 0
Esto no es tan sorprendente ya que agregar una constante a PAG 0 pero no PAG i no es realmente invariante de Lorentz.

Entonces, en mi opinión, la resta es más un truco que funciona, ya que solo calculamos realmente | o tu t | S | i norte | 2 por lo que no importa.

dos comentarios: 1. Su análisis se basa puramente en pQFT. ¿Existe alguna posibilidad de generalizar a una clase más amplia de QFT? (Sé que se ha argumentado que pQFT quizás sea el único QFT que tiene sentido). 2. De acuerdo con la mecánica cuántica (QM), se permite la representación proyectiva unitaria. La forma de obtener una representación proyectiva a partir de una representación lineal es cambiar el álgebra por una extensión de carga central (e imponer reglas de superselección si la topología del grupo de Lie no es trivial). ¿Cómo afecta este operador a su argumento?
Las representaciones proyectivas pueden surgir de extensiones centrales o topología no trivial de la variedad grupal. En el caso del grupo de Poincaré, puede demostrar que las representaciones proyectivas se obtienen considerando la cobertura universal, no se permite la extensión central (esto se hace en Weinberg). Si por pQFT te refieres a QFT perturbativo, entonces sí, mi análisis se basa únicamente en pQFT. Realmente no conozco mucho QFT no perturbativo que no involucre a SUSY, así que tal vez alguien más pueda responder eso.